LSAT Explanation PT 22, S4, Q26: A member of the British Parliament
LSAT Question Stem
Which one of the following, if true, most seriously weakens the argument attributed to the member of Parliament?
Logical Reasoning Question Type
This is a Weaken question.
Correct Answer
The correct answer to this question is D.
LSAT Question Complete Explanation
We have a Weaken question here, which means we are looking for an answer choice that undermines the conclusion of the argument or the relationship between the premises and the conclusion. Let's first analyze the passage and identify the premises and the conclusion.
Passage Analysis:
Premise 1: The first purpose of good social reform is to increase the sum total of human happiness.
Premise 2: Any reform which makes somebody happy is achieving its purpose.
Premise 3: The proposed reform would make the member of Parliament's constituents happy.
Conclusion: The proposed reform is a good social reform.
The argument's structure is based on the idea that if a reform makes someone happy, it is a good social reform. However, there seems to be a leap in logic here, as the initial premise is about increasing the sum total of human happiness, not just making someone happy.
An "Evaluate" question for this argument could be, "Does the proposed reform increase the sum total of human happiness?"
Now let's examine the answer choices:
a) Different things make different people happy.
- While this statement might be true, it does not directly address the argument's logic or the relationship between the premises and the conclusion. Therefore, this answer choice is incorrect.
b) The proposed reform would make a few people happy, but would not increase the happiness of most other people.
- This answer choice is tempting, but it does not necessarily weaken the argument. It only states that the happiness of others is not increased, which still allows for an increase in the sum total of happiness. Therefore, this answer choice is incorrect.
c) The proposed reform would affect only the member of Parliament's constituents and would make them happy.
- This answer choice actually strengthens the argument, as it supports the idea that the proposed reform would increase the happiness of the member's constituents. Therefore, this answer choice is incorrect.
d) Increasing some people's happiness might not increase the sum total of human happiness if others are made unhappy.
- This is the correct answer choice. It directly addresses the leap in logic mentioned earlier, pointing out that just because some people are made happy, it doesn't necessarily mean that the sum total of human happiness has increased. This weakens the argument.
e) Good social reforms usually have widespread support.
- The argument does not discuss widespread support, so this answer choice is irrelevant and incorrect.
In conclusion, the correct answer choice is (D), as it weakens the argument by questioning whether the proposed reform truly increases the sum total of human happiness.
